Question

In: Economics

Two firms compete by choosing price. Their demand functions are Q_1=20-P_1+P_2 and Q_2=20+P_1-P_2 where P_1 and...

Two firms compete by choosing price. Their demand functions are

Q_1=20-P_1+P_2 and Q_2=20+P_1-P_2

where P_1 and P_2 are the prices charged by each firm, respectively, and Q_1 and Q_2 are the resulting demands. Note that the demand for each good depends only on the difference in prices; if the two firms collude and set the same price, they could make that price as high as they wanted, and earn infinite profits. Marginal costs are zero.

a. Suppose the two firms set their prices at the same time. Find the resulting Nash equilibrium. What price will each firm charge, how much will it sell, and what will its profit be? (Hint: Maximize the profit of each firm with respect to its price.)

b. Suppose Firm 1 sets its price first and then Firm 2 sets its price. What price will each firm charge, how much will it sell, and what will its profit be?

c. Suppose you are one of these firms and that there are three ways you could play the game: (i) Both firms set price at the same time; (ii) You set price first; or (iii) Your competitor sets price first. If you could choose among these options, which would you prefer? Explain why.

Solutions

Expert Solution

Two firms compete by choosing price. Their demand functions are

Q_1=20-P_1+P_2 and Q_2=20+P_1-P_2

where P_1 and P_2 are the prices charged by each firm, respectively, and Q_1 and Q_2 are the resulting demands. Note that the demand for each good depends only on the difference in prices; if the two firms collude and set the same price, they could make that price as high as they wanted, and earn infinite profits. Marginal costs are zero.

Demand functions are :

Q1=20-p1 + pe
Q2= 20 - p2 + p1
MC = 0

a. the two firms set their prices at the same time. Find the resulting Nash equilibrium. What price will each firm charge, how much will it sell, and what will its profit be?

Bertrand price competition,
Firm 1 try to maximize profit , given p2
π1 = 20 p1 - p12 +p1p2
δπ2/δp1= 20 - 2p1+2p2=0
20+p2 = p1 (2)
P =20+p2 /2 = 10+1/2 p2
Similarly,from 2 wants to maximize profit ,given p
π2= 20p 2 - p22+ p1p2
δπ2/δp2= 20- 2p2 + p1 =0
( P2= 10+ 1/2 p1)
=: P2 = 10 + 1/2 ( 10+1/2 p2)
P2 - 1/ 4 p2 = 10+5 =15
P2 - 1/4 p2 = 10+5=15
P2 = 15× y/3

= 20
P1 = p2 = $ 20
Q1 = 20 - 20 +20

=20
Q2 =20
π1 = 20×20

= $ 400
π2 = $400

b. Suppose Firm 1 sets its price first and then Firm 2 sets its price. What price will each firm charge, how much will it sell, and what will its profit be?
:- Stackelberg price competition
We solve this by using backward induction, At stage 1 firm 2 try to maximize profit given P1
π2 = 20p2 - p22 + p1p2

δπ2/δp2 = 20 - 2p2 + p1 = 0

P2 = 10 + 1/2 p1

At stage 2, firm 1 observes p2 and then maximize profit

π1 = 20 p1 - p12 + p1p2

= 20 p1 - p12 + p1(10 + 1/2 p1)

= 20 p1 - p12 + 10 p1 + 1/2p12

= 30p1 - p12/2

δπ1/δp1 = 30- p1= 0

P1 = 30

P2 = 10 + 1/2(30)

= $25

Q1= 20-30+25

= 15

Q2 = 20+30-25

= 25

π1= 30(15)

= $450

π2 = 25(25)

=$625

There is a second mover advantage.

c. Suppose you are one of these firms and that there are three ways you could play the game: (i) Both firms set price at the same time; (ii) You set price first; or (iii) Your competitor sets price first. If you could choose among these options, which would you prefer? Explain why.

:-If I would be a firm , I will choose to set a price second or let my competitor sets price first, in by this I can maximize my profit($625) which is the that I can earn given this conditions.


Related Solutions

QUESTION 1 Two firms compete by choosing price. Their demand functions are Q1 = 20 -...
QUESTION 1 Two firms compete by choosing price. Their demand functions are Q1 = 20 - P1 +P2 and Q2 = 20 - P2 +P1 where P1 and P2 are the prices charged by each firm, respectively, and Q1 and Q2 are the resulting demands. Note that the demand for each good depends only on the difference in prices; if the two firms colluded and set the same price, they could make that price as high as they wanted, and...
Consider two firms that sell differentiated products and compete by choosing prices. Their demand functions are...
Consider two firms that sell differentiated products and compete by choosing prices. Their demand functions are Q1 = 72 – 3P1 + 2P2 and Q2 = 72 – 3P2 + 2P1 where P1 and P2 are the prices charged by firm 1 and 2, respectively, and Q1 and Q2 are the corresponding demands. All production costs are assumed to be zero. (a) Suppose the two firms set their prices simultaneously and non-cooperatively. Find the resulting Bertrand-Nash equilibrium. What price does...
Two firms compete in a market with inverse demand P(Q) = a − Q, where the...
Two firms compete in a market with inverse demand P(Q) = a − Q, where the aggregate quantity is Q = q1 + q2. The profit of firm i ∈ {1, 2} is πi(q1, q2) = P(Q)qi − cqi , where c is the constant marginal cost, with a > c > 0. The timing of the game is: (1) firm 1 chooses its quantity q1 ≥ 0; (2) firm 2 observes q1 and then chooses its quantity q2 ≥...
Two firms compete in a homogeneous product market where the inverse demand function is P =...
Two firms compete in a homogeneous product market where the inverse demand function is P = 10 -2Q (quantity is measured in millions). Firm 1 has been in business for one year, while Firm 2 just recently entered the market. Each firm has a legal obligation to pay one year’s rent of $0.5 million regardless of its production decision. Firm 1’s marginal cost is $2, and Firm 2’s marginal cost is $6. The current market price is $8 and was...
Two firms compete in a homogeneous product market where the inverse demand function is P =...
Two firms compete in a homogeneous product market where the inverse demand function is P = 20 -5Q (quantity is measured in millions). Firm 1 has been in business for one year, while Firm 2 just recently entered the market. Each firm has a legal obligation to pay one year’s rent of $1 million regardless of its production decision. Firm 1’s marginal cost is $2, and Firm 2’s marginal cost is $10. The current market price is $15 and was...
Two firms, Firm 1 and Firm 2, compete by simultaneously choosing prices. Both firms sell an...
Two firms, Firm 1 and Firm 2, compete by simultaneously choosing prices. Both firms sell an identical product for which each of 100 consumers has a maximum willingness to pay of $40. Each consumer will buy at most 1 unit, and will buy it from whichever firm charges the lowest price. If both firms set the same price, they share the market equally. Costs are given by ??(??)=16??ci(qi)=16qi. Because of government regulation, firms can only choose prices which are integer...
a.) Two identical firms compete as a Cournot duopoly. The market demand is P=100-2Q, where Q...
a.) Two identical firms compete as a Cournot duopoly. The market demand is P=100-2Q, where Q stands for the combined output of the two firms, Q=q1 +q2. The marginal cost for each firm is 4. Derive the best-response functions for these firms expressing what q1 and q2 should be. b.) Continuing from the previous question, identify the price and quantity that will prevail in the Cournot duopoly market c.) Now suppose two identical firms compete as a Bertrand duopoly. The...
Consider an industry with demand Q = a − p where 3 identical firms that compete...
Consider an industry with demand Q = a − p where 3 identical firms that compete a la Cournot. Each firm’s cost function is given by C = F + c q. Suppose two of the firms merge and that the merged firm’s cost function is given by C = F'+C'q, where F<F'<2F (a) Determine each firm’s market share before and after the merger. (b) Suppose that a = 10 and c = 3. Determine the Herfindahl index after the...
If firms compete by choosing quantities of output, is there an advantage to moving later or...
If firms compete by choosing quantities of output, is there an advantage to moving later or sooner? How might a firm commit to go sooner?
1. Consider a market of homogeneous products in which firms compete on price. Demand in this...
1. Consider a market of homogeneous products in which firms compete on price. Demand in this market is given by q(p) = 50 -10p Consumers buy from the producer with the lowest price. If the prices of both firms are the same then they purchase from E. There are both an incumbent firm M and a potential entrant E which can produce the good at marginal costs 3 and 2 , respectively. Prior to entry, E must incur an entry...
ADVERTISEMENT
ADVERTISEMENT
ADVERTISEMENT